Yim Yim

cho x;y;z >0 và \(x+y+z\ge1\). Chứng minh rằng

\(\frac{x^3}{y^2}+\frac{y^3}{z^2}+\frac{z^3}{x^2}\ge1\)

Đinh Đức Hùng
18 tháng 9 2017 lúc 12:43

Áp dụng bđt AM - GM ta có : 

\(\frac{x^3}{y^2}+x\ge2\sqrt{\frac{x^3}{y^2}.x}=\frac{2x^2}{y}\)

\(\frac{y^3}{z^2}+y\ge2\sqrt{\frac{y^3}{z^2}.y}=\frac{2y^2}{z}\)

\(\frac{z^3}{x^2}+z\ge2\sqrt{\frac{z^3}{x^2}.z}=\frac{2z^2}{x}\)

Cộng vế với vế ta được :

\(\frac{x^3}{y^2}+\frac{y^3}{z^2}+\frac{z^3}{x^2}+x+y+z\ge2\left(\frac{x^2}{y}+\frac{y^2}{z}+\frac{x^2}{z}\right)\)

Ta lại có : \(\left(\frac{x^2}{y}+\frac{y^2}{z}+\frac{x^2}{z}\right)\left(x+y+z\right)\ge\left(x+y+z\right)^2\)(bunhiacopxki)

\(\Rightarrow\frac{x^2}{y}+\frac{y^2}{z}+\frac{x^2}{z}\ge\frac{\left(x+y+z\right)^2}{x+y+z}=x+y+z\)

\(\Rightarrow\frac{x^3}{y^2}+\frac{y^3}{z^2}+\frac{z^3}{x^2}+x+y+z\ge2\left(\frac{x^2}{y}+\frac{y^2}{z}+\frac{x^2}{z}\right)\ge2\left(x+y+z\right)\)

\(\Rightarrow\frac{x^3}{y^2}+\frac{y^3}{z^2}+\frac{z^3}{x^2}\ge x+y+z\ge1\)(đpcm)

Bình luận (0)

Các câu hỏi tương tự
Nguyễn Đình Thiện
Xem chi tiết
Tôi Là Ai
Xem chi tiết
Hoàng Tử Lớp Học
Xem chi tiết
Nguyen Tuan Dung
Xem chi tiết
Siêu Nhân Lê
Xem chi tiết
Đinh Thị Ngọc Anh
Xem chi tiết
Pham Quoc Cuong
Xem chi tiết
Tôi Là Ai
Xem chi tiết
Tôi Là Ai
Xem chi tiết